HIV/IMMUNITY - 4th Semester

Réussis tes devoirs et examens dès maintenant avec Quizwiz!

1. To what is the increase in emerging and untreatable infections attributed (select all that apply)? a. The evolution of new infectious agents b. Use of antibiotics to treat viral infections c. Human population encroachment into wilderness areas d. Transmission of infectious agents from humans to animals e. An increased number of immunosuppressed and chronically ill people

1. a, b, c, e. Infectious agents, such as the human immunodeficiency virus (HIV) and hantavirus, have evolved to affect humans through closer association with animals as human populations push into wild animal habitats. The transfer of infectious agents from animals to humans has also resulted in West Nile virus and avian flu. Bacterial infections have also become untreatable as the result of genetic and biochemical changes stimulated by unnecessary or inadequate exposure to antibiotics. The increased number of immunosuppressed and chronically ill people also increase the emergence of untreatable infections.

1. Which type of immunity is the result of contact with the antigen through infection and is the longest lasting type of immunity? a. Active innate immunity c. Active acquired immunity b. Passive innate immunity d. Passive acquired immunity

1. c. Innate immunity is present at birth and its primary role is first-line defense against pathogens. Innate immunity is not antigen specific so it can respond within minutes to an invading microorganism without prior exposure to that organism.

1. The function of monocytes in immunity is related to their ability to a. stimulate the production of T and B lymphocytes. b. produce antibodies on exposure to foreign substances. c. bind antigens and stimulate natural killer cell activation. d. capture antigens by phagocytosis and present them to lymphocytes.

1. d,

10. Which characteristic describes immunoglobulin E (select all that apply)? a. Assists in parasitic infections b. Responsible for allergic reactions c. Present on the lymphocyte surface d. Assists in B-lymphocyte differentiation e. Predominant in secondary immune response f. Protects body surfaces and mucous membranes

10. a, b. Immunoglobulin E (IgE) assists in parasitic infections and causes allergic reactions. IgA protects body surfaces and mucous membranes. IgD assists in B-lymphocyte differentiation and is present on the lymphocyte surface. IgG is predominant in the secondary immune response.

10. Which statement about metabolic side e ects of ART is true (select all that apply)? a. ese are annoying symptoms that are ultimately harmless. b. ART-related body changes include central fat accumulation and peripheral wasting. c. Lipid abnormalities include increases in triglycerides and decreases in high-density cholesterol. d. Insulin resistance and hyperlipidemia can be treated with drugs to control glucose and cholesterol. e. Compared to uninfected people, insulin resistance and hyper- lipidemia are more di cult to treat in HIV-infected patients.

10. b, c, d,

10. What accurately describes rejection following transplantation? a. Hyperacute rejection can be treated with OKT3. b. Acute rejection can be treated with sirolimus or tacrolimus. c. Chronic rejection can be treated with tacrolimus or cyclosporine. d. Hyperacute reaction can usually be avoided if crossmatching is done before the transplantation.

10. d,

11. Which strategy can the nurse teach the patient to eliminate the risk of HIV transmission? a. Using sterile equipment to inject drugs b. Cleaning equipment used to inject drugs c. Taking zidovudine (AZT, ZDV, Retrovir) during pregnancy d. Using latex or polyurethane barriers to cover genitalia during sexual contact

11. a,

11. What are the important functions of cell-mediated immunity (select all that apply)? a. Fungal infections d. Contact hypersensitivity reactions b. Transfusion reactions e. Immunity against pathogens that survive outside cells c. Rejection of transplanted tissues

11. a, c, d. Functions of cell-mediated immunity include fungal infections, rejection of foreign tissue, contact hypersensitivity reactions, immunity against pathogens that survive inside cells, and destruction of cancer cells and tuberculosis. Transfusion reactions are from humoral immunity.

1. Emerging infections can a ect health care by (select all that apply) a. revealing antibiotic resistance. b. generating scienti c discoveries. c. creating a strain on limited resources. d. challenging established medical traditions. e. limiting travel options for nursing personnel.

1. a, b, c, d,

12. A 69-year-old woman asks the nurse whether it is possible to "catch" cancer because many of her friends of the same age have been diagnosed with different kinds of cancer. In responding to the woman, the nurse understands that what factor increases the incidence of tumors in older adults? a. An increase in autoantibodies c. Decreased differentiation of T lymphocytes b. Decreased activity of the bone marrow d. Decreased size and activity of the thymus gland

12. d. Aging has a pronounced effect on the thymus, which decreases in size and activity, leading to a decline in T cells and cell-mediated immunity. T-cell reduction is responsible for decreased tumor surveillance, resulting in an increase in cancer. B-cell activity also declines with advancing age but the bone marrow is relatively unaffected by increasing age. Circulating autoantibodies increase and are a factor in autoimmune diseases.

13. What describes the occurrence of a type IV or delayed hypersensitivity transplant reaction? a. Antigen links with specific IgE antibodies bound to mast cells or basophils releasing chemical mediators b. Cellular lysis or phagocytosis through complement activation following antigen-antibody binding on cell surfaces c. Sensitized T lymphocytes attack antigens or release cytokines that attract macrophages that cause tissue damage d. Antigens combined with IgG and IgM too small to be removed by mononuclear phagocytic system deposit in tissue and cause fixation of complement

13. c. When sensitized T lymphocytes attack antigens or release cytokines that attract macrophages and cause tissue damage, a type IV or delayed hypersensitivity reaction is occurring with transplant rejections as well as contact dermatitis, some drug sensitivity reactions, and hypersensitivity reactions to bacterial fungal and viral infections.

13. What finding supports the diagnosis of acquired immunodeficiency syndrome (AIDS) in the individual with HIV? a. Flu-like symptoms c. CD4+ T cells 200-500/μL b. Oral hairy leukoplakia d. Cytomegalovirus retinitis

13. d. Cytomegalovirus retinitis could be an opportunistic viral infection that occurs when AIDS is diagnosed. Flu-like symptoms occur in the acute HIV infection stage. CD4+ T cells drop to 200-500/μL and oral hairy leukoplakia are seen in the symptomatic infection stage of HIV.

14. What are examples of type I or IgE-mediated hypersensitivity reactions (select all that apply)? a. Asthma b. Urticaria c. Angioedema d. Allergic rhinitis e. Atopic dermatitis f. Contact dermatitis g. Anaphylactic shock h. Transfusion reactions i. Goodpasture syndrome

14. a, b, c, d, e, g. These are the anaphylactic or atopic responses that can be seen with a type I or IgE-mediated hypersensitivity reaction to specific allergens. Contact dermatitis is seen with a type IV or delayed hypersensitivity reaction. Transfusion reactions and Goodpasture syndrome are seen with a type II or cytotoxic hypersensitivity reaction.

14. Why do opportunistic diseases develop in an individual with AIDS? a. They are side effects of drug treatment of AIDS. b. They are sexually transmitted to individuals during exposure to HIV. c. They are characteristic in individuals with stimulated B and T lymphocytes. d. These infections or tumors occur in a person with an incompetent immune system.

14. d. Organisms that are nonvirulent or that cause limited or localized diseases in an immunocompetent person can cause severe, debilitating, and life-threatening infections in persons with impaired immune function.

17. A patient comes to the clinic and requests testing for HIV infection. Before administering testing, what is most important for the nurse to do? a. Ask the patient to identify all sexual partners b. Determine when the patient thinks exposure to HIV occurred c. Explain that all test results must be repeated at least twice to be valid d. Discuss prevention practices to prevent transmission of the HIV to others

17. b. Because there is a median delay of several weeks after infection before antibodies can be detected, testing during this "window" may result in false-negative results. Risky behaviors that may expose a person to HIV should be discussed and possible scheduling for repeated testing done. Positive results on initial testing will be verified by additional testing. Identification of sexual partners and prevention practices are important but do not relate immediately to the testing situation.

23. Why is plasmapheresis indicated in the treatment of autoimmune disorders? a. Obtain plasma for analysis and evaluation of specific autoantibodies b. Decrease high lymphocyte levels in the blood to prevent immune responses c. Remove autoantibodies, antigen-antibody complexes, and inflammatory mediators of immune reactions d. Add monocytes to the blood to promote removal of immune complexes by the mononuclear phagocyte system

23. c. Plasmapheresis is the removal of plasma from the blood and in autoimmune disorders is used to remove pathogenic substances found in plasma, such as autoantibodies, antigen-antibody complexes, and inflammatory mediators. Circulating blood cells are not affected by plasmapheresis, nor are blood cells added.

4. A patient with diarrhea has been diagnosed with Clostridium difficile. Along with standard precautions, which kind of transmission-based precautions will be used when the nurse is caring for this patient? a. Droplet precautions c. Isolation precautions b. Contact precautions d. Airborne precautions

4. b. Contact precautions are used with standard precautions when microorganisms can be transmitted by direct patient contact.

4. During HIV infection a. the virus replicates mainly in B-cells before spreading to CD4+ T cells. b. infection of monocytes may occur, but antibodies quickly destroy these cells. c. the immune system is impaired predominantly by the eventual widespread destruction of CD4+ T cells. d. a long period of dormancy develops during which HIV cannot be found in the blood and there is little viral replication.

4. c,

4. In a type I hypersensitivity reaction the primary immunologic disorder appears to be a. binding of IgG to an antigen on a cell surface. b. deposit of antigen-antibody complexes in small vessels. c. release of cytokines used to interact with speci c antigens. d. release of chemical mediators from IgE-bound mast cells and basophils.

4. d

4. Which T lymphocytes are involved in direct attack and destruction of foreign pathogens? a. Dendritic cells c. T helper (CD4) cells b. Natural killer cells d. T cytotoxic (CD8) cells

4. d. T cytotoxic cells directly attack antigens on the cell membrane of foreign pathogens and release cytolytic substances that destroy pathogens. Dendritic cells primarily capture antigens at sites of contact with the external environment and then transport the antigen to a T cell with specificity for the antigen. Natural killer cells are involved in cell-mediated immunity but are not considered T lymphocytes. CD4 cells (T helper cells) are involved in the regulation of cell-mediated immunity and humoral antibody response.

A patient is to start on efavirenz (Sustiva). Which points are important for the nurse to include in health teaching for this patient? (Select all that apply.) a. Dose is given at bedtime to minimize CNS adverse effects. b. Avoid alcohol because of adverse effects to the liver. c. Dose is given after breakfast to minimize CNS adverse effects. d. High-fat meals will decrease absorption of medication. e. Hyperglycemia, jaundice, and diabetes mellitus are side effects.

5,a,b

7. Where and into what do activated B lymphocytes differentiate? a. Spleen; natural killer cells that destroy infected cells b. Bone marrow; plasma cells that secrete immunoglobulins c. Thymus; memory B-cells that retain a memory of the antigen d. Bursa of Fabricius; helper cells that in turn activate additional B lymphocytes

7. b. B lymphocytes activated in the bone marrow by the presentation of an antigen differentiate into many plasma cells that secrete immunoglobulins and only a few memory cells that retain recognition of the antigen as foreign. Helper cells are T lymphocytes and natural killer cells are large, granular lymphocytes that are neither B nor T lymphocytes. The spleen filters foreign substances from the blood. The thymus differentiates T lymphocytes. The bursa of Fabricius is found in birds, not humans.

7. Screening for HIV infection generally involves a. laboratory analysis of blood to detect HIV antigen. b. electrophoretic analysis for HIV antigen in plasma. c. laboratory analysis of blood to detect HIV antibodies. d. analysis of lymph tissues for the presence of HIV RNA.

7. c,

7. Association between HLA antigens and diseases is most com- monly found in what disease conditions? a. Malignancies b. Infectious diseases c. Neurologic diseases d. Autoimmune disorders

7. d,

A patient who is infected with human immunodeficiency virus (HIV) is being taught by the nurse about health promotion activities such as good nutrition; avoiding alcohol, tobacco, drug use, and exposure to infectious agents; keeping up to date with vaccines; getting adequate rest; and stress management. What is the rationale behind these interventions that the nurse knows? A delaying disease progression B preventing disease transmission C helping to cure HIV infection D Enabling an increase in self-care activities

A

An older adult patient is brought to the primary health care provider by an adult child reporting confusion. What testing should the nurse anticipate obtaining from this patient? A Urinalysis B Sputum culture C Red blood cell count D White blood cell count

A

The nurse teaches the staff ensuring that standard precautions should be used when providing care for which type of patient? A All patients regardless of diagnosis B Pediatric and gerontologic patients C Patients who are immunocompromised D Patients with a history of infectious diseases

A

When teaching a patient infected with HIV regarding transmission of the virus to others, which statement made by the patient would indicate a need for further teaching? A "I will need to isolate any tissues I use so as not to infect my family." B "I will notify all of my sexual partners so they can get tested for HIV." C "Unprotected sexual contact is the most common mode of transmission." D "I do not need to worry about spreading this virus to others by sweating at the gym."

A

What should the nurse teach the patients in the assisted living facility to decrease their risk for antibiotic- resistant infection (select all that apply.)? Select all that apply. A Wash hands frequently. B Take antibiotics as prescribed. C Take the antibiotic until it is gone. D Take antibiotics to prevent illnesses like colds. Save E leftover antibiotics to take if needed later.

A B C

13. After a urinary diversion pancreatic transplant, the nurse must anticipate which order? a. Continuous bladder irrigation b. Intermittent insulin injections c. Removal of the nasogastric tube as soon as the patient is alert d. Daily hematocrit and hemoglobin levels.

ANS: A An increased potential for urinary catheter occlusion exists for pancreas recipients who have undergone a urinary diversion procedure. The exocrine pancreatic enzymes make the urine more viscous, and they irritate the anastomosis site on the bladder, causing an increased risk of bleeding. Gentle aspiration techniques can be used to clear out the clots. Continuous bladder irrigation may be necessary to keep the catheter patent. A nasogastric tube is usually placed and remains for 24 to 48 hours after surgery. A continuous insulin drip may be used to rest the new graft and prevent hyperglycemia.

15. The nurse teaches a patient about drug therapy after a kidney transplant. Which statement by the patient would indicate a need for further instructions? a. "After a couple of years, it is likely that I will be able to stop taking the cyclosporine." b. "If I develop an acute rejection episode, I will need to have other types of drugs given IV." c. "I need to be monitored closely because I have a greater chance of developing malignant tumors." d. "The drugs are given in combination because they inhibit different ways the kidney can be rejected."

ANS: A Cyclosporine, a calcineurin inhibitor, will need to be continued for life. The other patient statements are accurate and indicate that no further teaching is necessary about those topics.

6. Who determines the medical suitability of the patient for organ donation? a. The organ procurement organization (OPO) coordinator b. The patient's family c. The admitting health care provider d. The transplant team

ANS: A Determining medical suitability is solely the responsibility of the OPO. Speaking to the family about donation is also the responsibility of the OPO unless designated requestors at the hospital have been trained to do so.

19. Which of the following nursing diagnoses would be appropriate for a patient who has received a kidney transplant? a. Deficient fluid volume c. Urinary retention b. Excess fluid volume d. Decreased cardiac output

ANS: A Hypovolemia can lead to compromised blood flow to the kidney, acute kidney injury, and possible graft failure. The new kidney will be producing large amounts of urine, and fluid replacement, usually maintained in a 1:1 ratio, must be sustained.

11. The longest and most difficult part of the recipient hepatectomy is a. stage 1. c. stage 3. b. stage 2. d. grade 1.

ANS: A Liver transplantation surgery can be divided into three stages: (1) recipient hepatectomy (the longest and most difficult part of the surgery), (2) vascular anastomoses with donor liver, and (3) biliary anastomosis. Grade 1 refers to living liver donor complications.

16. A nurse is caring for a patient who is receiving sirolimus. The nurse knows to monitor the patient for a. bleeding. c. rigors. b. diarrhea. d. hypotension.

ANS: A Other primary side effects of this medication include hyperlipidemia and myelosuppression. Most of the myelosuppressive effect is directed at platelets, and severe thrombocytopenias can result, making it necessary to discontinue the medication.

14. Which statement made by a patient who has had a urinary diversion pancreas transplant indicates the need for further teaching? a. "I no longer need to monitor my blood glucose levels." b. "I will need to have periodic pancreas biopsies." c. "I may feel the urge to urinate frequently." d. "I will give urine samples to monitor for rejection."

ANS: A Patients with functional pancreas grafts continue to need glucose monitoring at home but often forget to continue this practice after they no longer require insulin. Continued monitoring with frequent clinic visits is required for several months after pancreas transplantation.

6. Which patient exposure by the nurse is most likely to require postexposure prophylaxis when the patient's human immunodeficiency virus (HIV) status is unknown? a. Needle stick with a needle and syringe used to draw blood b. Splash into the eyes when emptying a bedpan containing stool c. Contamination of open skin lesions with patient vaginal secretions d. Needle stick injury with a suture needle during a surgical procedur

ANS: A Puncture wounds are the most common means for workplace transmission of blood-borne diseases, and a needle with a hollow bore that had been contaminated with the patient's blood would be a high-risk situation. The other situations described would be much less likely to result in transmission of the virus.

24. The Model for End-Stage Liver Disease (MELD) formula is used to a. calculate risk of 3-month mortality in patients 12 years old or older. b. calculate risk of 3-month mortality in patients 11 years or younger. c. calculate risk of 6-month mortality in patient 18 years or older. d. calculate risk of 6-month mortality in patient 17 years or younger.

ANS: A The Model for End-Stage Liver Disease (MELD) formula is used in all U.S. transplant centers to calculate the risk of 3-month mortality in patients 12 years old or older. The Pediatric End-Stage Liver Disease (PELD) formula is used to calculate risk of 3-month mortality for patients 11 years old or younger.

11. Which information would be most important to help the nurse determine if the patient needs human immunodeficiency virus (HIV) testing? a. Patient age b. Patient lifestyle c. Patient symptoms d. Patient sexual orientation

ANS: A The current Center for Disease Control (CDC) policy is to offer routine testing for HIV to all individuals age 13 to 64. Although lifestyle, symptoms, and sexual orientation may suggest increased risk for HIV infection, the goal is to test all individuals in this age range.

17. To evaluate the effectiveness of antiretroviral therapy (ART), which laboratory test result will the nurse review? a. Viral load testing b. Enzyme immunoassay c. Rapid HIV antibody testing d. Immunofluorescence assay

ANS: A The effectiveness of ART is measured by the decrease in the amount of virus detectable in the blood. The other tests are used to detect HIV antibodies, which remain positive even with effective ART.

3. A patient is being evaluated for possible atopic dermatitis. The nurse expects elevation of which laboratory value? a. IgE b. IgA c. Basophils d. Neutrophils

ANS: A Serum IgE is elevated in an allergic response (type 1 hypersensitivity disorders). The eosinophil level will be elevated rather than neutrophil or basophil counts. IgA is located in body secretions and would not be tested when evaluating a patient who has symptoms of atopic dermatitis.

5. Which of the following is an anticipated finding after cardiac transplantation? a. Two P waves on the electrocardiogram (ECG) b. High cardiac output c. Anginal pain d. Resting heart rate of 60 to 70 beats/min

ANS: A An ECG abnormality unique to a transplanted heart is the presence of a second P wave generated by the native sinoatrial (SA) node left in the atrial cuff. Because this impulse does not cross the suture line, it is capable of conducting only through the remnant of the native recipient atria. However, this is not seen in hearts transplanted using the bicaval technique because the native right atrium and therefore the SA node is removed.

5. A patient who collects honey to earn supplemental income has developed a hypersensitivity to bee stings. Which statement, if made by the patient, would indicate a need for additional teaching? a. "I need to find another way to earn extra money." b. "I will get a prescription for epinephrine and learn to self-inject it." c. "I will plan to take oral antihistamines daily before going to work." d. "I should wear a Medic-Alert bracelet indicating my allergy to bee stings."

ANS: C Because the patient is at risk for bee stings and the severity of allergic reactions tends to increase with added exposure to allergen, taking oral antihistamines will not adequately control the patient's hypersensitivity reaction. The other patient statements indicate a good understanding of management of the problem.

12. Which patient should the nurse assess first? a. Patient with urticaria after receiving an IV antibiotic b. Patient who has graft-versus-host disease and severe diarrhea c. Patient who is sneezing after having subcutaneous immunotherapy d. Patient with multiple chemical sensitivities who has muscle stiffness

ANS: C Sneezing after subcutaneous immunotherapy may indicate impending anaphylaxis and assessment and emergency measures should be initiated. The other patients also have findings that need assessment and intervention by the nurse, but do not have evidence of life-threatening complications.

11. Which statement by a patient would alert the nurse to a possible immunodeficiency disorder? a. "I take one baby aspirin every day to prevent stroke." b. "I usually eat eggs or meat for at least 2 meals a day." c. "I had my spleen removed many years ago after a car accident." d. "I had a chest x-ray 6 months ago when I had walking pneumonia."

ANS: C Splenectomy increases the risk for septicemia from bacterial infections. The patient's protein intake is good and should improve immune function. Daily aspirin use does not affect immune function. A chest x-ray does not have enough radiation to suppress immune function.

1. Patients meeting criteria for imminent death and cardiac death are to be a. immediately placed on life support and referred to the organ procurement organizations. b. referred to the organ procurement organizations at the time of their death. c. referred to the organ procurement organizations in a timely manner. d. referred to physician to make the decision if they are a candidate for organ donation.

ANS: C The Centers for Medicare and Medicaid Services guidelines, The Joint Commission standards, and hospital policies require that patients meeting criteria for imminent death and cardiac death be referred to an organ procurement organization in a timely manner.

21. An older adult who takes medications for coronary artery disease has just been diagnosed with asymptomatic chronic human immunodeficiency virus (HIV) infection. Which information will the nurse include in patient teaching? a. Many medications have interactions with antiretroviral drugs. b. Less frequent CD4+ level monitoring is needed in older adults. c. Hospice care is available for patients with terminal HIV infection. d. Progression of HIV infection occurs more rapidly in older patients.

ANS: A The nurse will teach the patient about potential interactions between antiretrovirals and the medications that the patient is using for chronic health problems. Treatment and monitoring of HIV infection is not affected by age. A patient with asymptomatic HIV infection is not a candidate for hospice. Progression of HIV is not affected by age, although it may be affected by chronic disease.

20. Which of these patients being seen at the human immunodeficiency virus (HIV) clinic should the nurse assess first? + a. Patient whose latest CD4 count is 250/μL b. Patient whose rapid HIV-antibody test is positive c. Patient who has had 10 liquid stools in the last 24 hours d. Patient who has nausea from prescribed antiretroviral drugs

ANS: C The nurse should assess the patient for dehydration and hypovolemia. The other patients also will require assessment and possible interventions, but do not require immediate action to prevent complications such as hypovolemia and shock.

13. Ten days after receiving a bone marrow transplant, a patient develops a skin rash. What would the nurse suspect is the cause of this patient's skin rash? a. The donor T cells are attacking the patient's skin cells. b. The patient's antibodies are rejecting the donor bone marrow. c. The patient is experiencing a delayed hypersensitivity reaction. d. The patient will need treatment to prevent hyperacute rejection.

ANS: A The patient's history and symptoms indicate that the patient is experiencing graft-versus-host disease, in which the donated T cells attack the patient's tissues. The history and symptoms are not consistent with rejection or delayed hypersensitivity.

4. An older adult patient who is having an annual check-up tells the nurse, "I feel fine, and I don't want to pay for all these unnecessary cancer screening tests!" Which information should the nurse plan to teach this patient? a. Consequences of aging on cell-mediated immunity b. Decrease in antibody production associated with aging c. Impact of poor nutrition on immune function in older people d. Incidence of cancer-stimulating infections in older individuals

ANS: A The primary impact of aging on immune function is on T cells, which are important for immune surveillance and tumor immunity. Antibody function is not affected as much by aging. Poor nutrition can also contribute to decreased immunity, but there is no evidence that it is a contributing factor for this patient. Although some types of cancer are associated with specific infections, this patient does not have an active infection.

2. Which of the statements would indicate that a patient who has received an organ transplant understands the teaching about his or her immunosuppressive medications? a. "My drug dosages will be lower because the medications enhance each other." b. "I will be less prone to side effects because I will be taking more than one drug." c. "Lower doses of these medications put me at greater risk for infection." d. "Taking more than one medication will put me at risk for developing allergies."

ANS: A These "triple-drug" regimens are designed to prevent rejection while reducing the toxicity of the individual medications.

20. Because a patient who has received a lung transplant is at risk for ventilation/perfusion (V/Q) mismatch, it would be a priority for the nurse to monitor a. blood gases. c. lung sounds. b. cardiac output. d. blood pressure.

ANS: A V/Q mismatch causes hypoxemia and is monitored with blood gases.

8. Which of the following goals would be a priority after lung transplantation? a. Wean the patient from the ventilator. c. Start corticosteroid therapy. b. Maintain hypotensive levels. d. Initiate pulmonary function studies.

ANS: A Ventilation settings are determined partially by the underlying disease process and patient progression. Regular suctioning of secretions is crucial to maintain airway clearance. Extubation is performed after satisfactory gas exchange and lung mechanics are accomplished and most patients are extubated within 24 to 48 hours. Evaluation for graft dysfunction, reperfusion injury, gas trapping, and phrenic nerve injury is ongoing. Early mobilization after extubation is essential for improved pulmonary toilet and helps prevent reintubation.

22. The registered nurse (RN) caring for an HIV-positive patient admitted with tuberculosis can delegate which action to unlicensed assistive personnel (UAP)? a. Teach the patient about how to use tissues to dispose of respiratory secretions. b. Stock the patient's room with all the necessary personal protective equipment. c. Interview the patient to obtain the names of family members and close contacts. d. Tell the patient's family members the reason for the use of airborne precautions.

ANS: B A patient diagnosed with tuberculosis would be placed on airborne precautions. Because all health care workers are taught about the various types of infection precautions used in the hospital, the UAP can safely stock the room with personal protective equipment. Obtaining contact information and patient teaching are higher-level skills that require RN education and scope of practice.

22. A clinic patient is experiencing an allergic reaction to an unknown allergen. Which action is most appropriate for the registered nurse (RN) to delegate to a licensed practical/vocational nurse (LPN/LVN)? a. Perform a focused physical assessment. b. Obtain the health history from the patient. c. Teach the patient about the various diagnostic studies. d. Administer skin testing by the cutaneous scratch method.

ANS: D LPN/LVNs are educated and licensed to administer medications under the supervision of an RN. RN-level education and the scope of practice include assessment of health history, focused physical assessment, and patient teaching.

10. A patient with liver failure manifested by portal hypertension and ascites is hospitalized and waiting for a liver transplant. Which of the following orders should the nurse question? a. Low-sodium diet b. Low-protein diet c. ABO typing d. Human leukocytic antigen tissue typing

ANS: D Liver failure is managed with a low-protein diet to decrease the risk of hepatic encephalopathy. A low-sodium diet is ordered to help manage the ascites. Blood type and body size are used to match a liver donor, not HLA testing.

4. A patient who is diagnosed with acquired immunodeficiency syndrome (AIDS) tells the nurse, "I feel obsessed with thoughts about dying. Do you think I am just being morbid?" Which response by the nurse is best? a. "Thinking about dying will not improve the course of AIDS." b. "It is important to focus on the good things about your life now." c. "Do you think that taking an antidepressant might be helpful to you?" d. "Can you tell me more about the kind of thoughts that you are having?"

ANS: D More assessment of the patient's psychosocial status is needed before taking any other action. The statements, "Thinking about dying will not improve the course of AIDS" and "It is important to focus on the good things in life" discourage the patient from sharing any further information with the nurse and decrease the nurse's ability to develop a trusting relationship with the patient. Although antidepressants may be helpful, the initial action should be further assessment of the patient's feelings.

10. The nurse is caring for a patient undergoing plasmapheresis. The nurse should assess the patient for which clinical manifestation? a. Shortness of breath b. High blood pressure c. Transfusion reaction d. Numbness and tingling

ANS: D Numbness and tingling may occur as the result of the hypocalcemia caused by the citrate used to prevent coagulation. The other clinical manifestations are not associated with plasmapheresis.

15. Which statement made by a patient who has received an organ transplant indicates that the teaching was effective? a. "I will finally be able to eat a regular diet." b. "I will establish a routine for checking any skin changes." c. "I will check my blood pressure regularly to make sure it's not too high." d. "After my drug regimen is established, I won't have to worry about rejection."

ANS: B Immunosuppressed patients are at greater risk for developing skin cancer. After transplantation, such a patient needs to monitor for hypotension and eat a diet that is low in sodium, fat, and cholesterol.

16. The nurse prepares to administer the following medications to a hospitalized patient with human immunodeficiency (HIV). Which medication is most important to administer at the right time? a. Oral acyclovir (Zovirax) b. Oral saquinavir (Invirase) c. Nystatin (Mycostatin) tablet d. Aerosolized pentamidine (NebuPent)

ANS: B It is important that antiretrovirals be taken at the prescribed time every day to avoid developing drug-resistant HIV. The other medications should also be given as close as possible to the correct time, but they are not as essential to receive at the same time every day.

5. A pregnant woman with a history of asymptomatic chronic human immunodeficiency virus (HIV) infection is seen at the clinic. The patient states, "I am very nervous about making my baby sick." Which information will the nurse include when teaching the patient? a. The antiretroviral medications used to treat HIV infection are teratogenic. b. Most infants born to HIV-positive mothers are not infected with the virus. c. Because she is at an early stage of HIV infection, the infant will not contract HIV. d. It is likely that her newborn will become infected with HIV unless she uses antiretroviral therapy (ART).

ANS: B Only 25% of infants born to HIV-positive mothers develop HIV infection, even when the mother does not use ART during pregnancy. The percentage drops to 2% when ART is used. Perinatal transmission can occur at any stage of HIV infection (although it is less likely to occur when the viral load is lower). ART can safely be used in pregnancy, although some ART drugs should be avoided.

4. According to the critical pathway developed by United Network for Organ Sharing, which of the following should be carried out by the critical care nurse? (Select all that apply.) a. Complete the operating room checklist. b. Document the date and time of brain death. c. Insert two large-bore IV catheters. d. Maintain patient temperature between 36.5° and 37.5° C. e. Notify the supervisor of pending donation.

ANS: B, C, D Each member of the health care team must work together to prepare the donor.

3. Many barriers exist to increasing the number of organ donors to meet today's growing need. Which of the following are barriers related to hospital factors? (Select all that apply.) a. Limited resources b. Lack of interest c. Failure to approve protocols d. Nurses' attitudes e. Delay in termination of life support

ANS: B, C, D, E A recent study of hospital donation practices and their impact on organ donation outcomes revealed gaps in knowledge of organ donation; brain death; referral criteria; and at times, a poor relationship between the hospital and the organ procurement organization. It is important that nurses are knowledgeable about the organ donation process. Nurses must assess their own beliefs that pertain to organ donation because the attitude of the nurse and care given to the family can impact the outcome of the donation.

2. Not every patient with end-stage liver disease is a candidate for receiving a transplant. Which of the following conditions are contraindications to a kidney transplant? (Select all that apply.) a. Malignancy during the past 5 years b. Active infectious process c. Advanced cardiopulmonary disease d. Recreational drug use e. Nonadherence to current medical regimen

ANS: B, C, D, E Contraindications to kidney transplantation include malignancy during the past 3 years, active infectious process, advanced cardiopulmonary disease, high risk for surgery, nonadherence to current medical regimen, recreational drug use, and other serious contributing disease processes.

2. According to the Center for Disease Control (CDC) guidelines, which personal protective equipment will the nurse put on when assessing a patient who is on contact precautions for diarrhea caused by Clostridium difficile (select all that apply)? a. Mask b. Gown c. Gloves d. Shoe covers e. Eye protection

ANS: B,C Because the nurse will have substantial contact with the patient and bedding when doing an assessment, gloves and gowns are needed. Eye protection and masks are needed for patients in contact precautions only when spraying or splashing is anticipated. Shoe covers are not recommended in the CDC guidelines.

3. A patient with a positive rapid antibody test result for human immunodeficiency virus (HIV) is anxious and does not appear to hear what the nurse is saying. What action by the nurse is most important at this time? a. Teach the patient about the medications available for treatment. b. Inform the patient how to protect sexual and needle-sharing partners. c. Remind the patient about the need to return for retesting to verify the results. d. Ask the patient to notify individuals who have had risky contact with the patient.

ANS: C After an initial positive antibody test, the next step is retesting to confirm the results. A patient who is anxious is not likely to be able to take in new information or be willing to disclose information about HIV status of other individuals.

15. A patient treated for human immunodeficiency virus (HIV) infection for 6 years has developed fat redistribution to the trunk, with wasting of the arms, legs, and face. What instructions will the nurse give to the patient? a. Review foods that are higher in protein. b. Teach about the benefits of daily exercise. c. Discuss a change in antiretroviral therapy. d. Talk about treatment with antifungal agents.

ANS: C A frequent first intervention for metabolic disorders is a change in antiretroviral therapy (ART). Treatment with antifungal agents would not be appropriate because there is no indication of fungal infection. Changes in diet or exercise have not proven helpful for this problem.

20. A patient who is anxious and has difficulty breathing seeks treatment after being stung by a wasp. What is the nurse's priority action? a. Have the patient lie down. b. Assess the patient's airway. c. Administer high-flow oxygen. d. Remove the stinger from the site.

ANS: B The initial action with any patient with difficulty breathing is to assess and maintain the airway. The other actions also are part of the emergency management protocol for anaphylaxis, but the priority is airway maintenance.

9. Which of the following would indicate that a newly transplanted liver is not functioning properly? a. Increased serum glucose b. Low aspartate aminotransferase (AST) and alanine aminotransferase (ALT) c. Elevated prothrombin time d. Decreased blood urea nitrogen and creatinine

ANS: C A patient with suspected primary nonfunction of a liver graft demonstrates (1) hemodynamic instability, (2) progressive deterioration of kidney function, (3) coagulopathies and abnormal serum liver function laboratory test results, (4) hypoglycemia, (5) continued ventilatory dependence, and (6) an inability to awaken from anesthesia.

14. An adolescent patient seeks care in the emergency department after sharing needles for heroin injection with a friend who has hepatitis B. To provide immediate protection from infection, what medication will the nurse administer? a. Corticosteroids b. Gamma globulin c. Hepatitis B vaccine d. Fresh frozen plasma

ANS: B The patient should first receive antibodies for hepatitis B from injection of gamma globulin. The hepatitis B vaccination series should be started to provide active immunity. Fresh frozen plasma and corticosteroids will not be effective in preventing hepatitis B in the patient.

19. A patient who has received allergen testing using the cutaneous scratch method has developed itching and swelling at the skin site. Which action should the nurse take first? a. Administer epinephrine. b. Apply topical hydrocortisone. c. Monitor the patient for lower extremity edema. d. Ask the patient about exposure to any new lotions or soaps.

ANS: A The initial symptoms of anaphylaxis are itching and edema at the site of the exposure. Hypotension, tachycardia, dilated pupils, and wheezes occur later. Rapid administration of epinephrine when excessive itching or swelling at the skin site is observed can prevent the progression to anaphylaxis. Topical hydrocortisone would not deter an anaphylactic reaction. Exposure to lotions and soaps does not address the immediate concern of a possible anaphylactic reaction. The nurse should not wait and observe for edema. The nurse should act immediately in order to prevent progression to anaphylaxis.

13. Which nursing action will be most useful in assisting a college student to adhere to a newly prescribed antiretroviral therapy (ART) regimen? a. Give the patient detailed information about possible medication side effects. b. Remind the patient of the importance of taking the medications as scheduled. c. Encourage the patient to join a support group for students who are HIV positive. d. Check the patient's class schedule to help decide when the drugs should be taken.

ANS: D The best approach to improve adherence is to learn about important activities in the patient's life and adjust the ART around those activities. The other actions also are useful, but they will not improve adherence as much as individualizing the ART to the patient's schedule.

14. A patient with human immunodeficiency virus (HIV) infection has developed Mycobacterium avium complex infection. Which outcome would be appropriate for the nurse to include in the plan of care? a. The patient will be free from injury. b. The patient will receive immunizations. c. The patient will have adequate oxygenation. d. The patient will maintain intact perineal skin.

ANS: D The major manifestation of M. avium infection is loose, watery stools, which would increase the risk for perineal skin breakdown. The other outcomes would be appropriate for other complications (pneumonia, dementia, influenza, etc.) associated with HIV infection.

A nurse was accidently stuck with a needle used on a patient who is infected with human immunodeficiency virus (HIV). After reporting the incident, what care should this nurse first receive? A Personal protective equipment B Combination antiretroviral therapy C Counseling to report blood exposures D A negative evaluation by the manager

B

A parent does not want their child to have any extra immunizations for diseases that no longer occur. What teaching about immunization should the nurse provide this mother? A There is currently no need for those older vaccines. B There is a reemergence of some of the infections, such as pertussis. C There is no longer an immunization available for some of those diseases. D The only way to protect your child is to have the federally required vaccines.

B

The nurse has experienced a recent increase in the incidence of hospital care-associated infections (HAIs) on the unit. Which nursing action should be prioritized in the response to this trend? A Use of gloves during patient contact B Frequent and thorough hand washing C Prophylactic, broad-spectrum antibiotics D Fitting and appropriate use of N95 masks

B

The nurse is caring for a patient newly diagnosed with human immunodeficiency virus (HIV). What does the nurse explain to the patient the criteria for diagnosis is based on? A Presence of HIV antibodies B CD4+ T cell count below 200/μL C Presence of oral hairy leukoplakia D White blood cell count below 5000/μL

B

The nurse is teaching a group of young adults who live in a dormitory about the prevention of antibiotic- resistant infections. What should be included in the teaching plan? A Save leftover antibiotics for future uses. B Hand washing can prevent many infections. C Antibiotics are indicated for preventing most colds. D Stop taking prescribed antibiotics when symptoms improve.

B

The patient is diagnosed with vancomycin-resistant enterococci (VRE) infection in a surgical wound. What infection precautions should the nurse use to best prevent transmission of the infection to others? A Droplet precautions B Contact precautions C Airborne precautions D Standard precautions

B

A patient was exposed to human immunodeficiency virus (HIV) 2 weeks ago through sharing needles with other substance users. What symptoms will the nurse teach the patient to report that would indicate the patient has developed an acute HIV infection? Cough, diarrhea, headaches, blurred vision, muscle fatigue A Night sweats, fatigue, fever, and persistent B generalized lymphadenopathy C oropharyngeal candidiasis or thrush, vaginal candidate infection, or oral or genital herpes D Flu-like symptoms such as fever, sore throat, swollen lymph glands, nausea, or vomiting.

D

A patient is admitted to the emergency department (ED) with fever, swollen lymph glands, sore throat, headache, malaise, joint pain, and diarrhea. What nursing actions will help identify the need for further assessment of the cause of this patient's manifestations (select all that apply.)? A assessment of lung sounds B sexual behavior C living conditions D drug and syringe use E exposure to an ill person

B D

A pregnant woman who was tested and diagnosed with human immunodeficiency virus (HIV) infection is very upset. What should the nurse teach this patient about her baby's risk of being born with HIV infection? A "The baby will probably be infected with HIV." B "Only an abortion will keep your baby from having HIV." C "Treatment with antiretroviral therapy will decrease the baby's chance of HIV infection." D "The duration and freq of contact with organism will determine if the baby gets HIV"

C

The nurse is monitoring the effectiveness of antiretroviral therapy (ART) for a patient with acquired immunodeficiency syndrome (AIDS). What laboratory study result indicates the medications are effective? A Increased viral load B Decreased neutrophil count C Increased CD4+ T cell count D Decreased WBC count

C

The nurse is providing care for a patient who has been living with human immunodeficiency virus (HIV) for several years. Which assessment finding most clearly indicates an acute exacerbation of the disease? A A new onset of polycythemia B Presence of mononucleosis-like symptoms C A sharp decrease in the patient's CD4+ count D A sudden incase in the patient's WBC count

C

The nurse is providing postoperative care for a patient with human immunodeficiency virus (HIV) infection after an appendectomy. What type of precautions should the nurse observe to prevent the transmission of this disease? A Droplet precautions B Contact precautions C Airborne precautions D Standard precautions

C

A heterosexual patient is concerned that they may contract human immunodeficiency virus (HIV) from a bisexual partner. What should the nurse include when teaching about preexposure prophylaxis (select all that apply.)? A Take fluconazole (Diflucan). B Take amphotericin B (Fungizone). C Use condoms for risk-reducing sexual relations. D take emtricitabine and tenofovir (truvada) regularly E Have regular HIV testing for herself and her husband

C D E

During routine prenatal testing, a patient is diagnosed with HIV infection. To help prevent perinatal transmis- sion of HIV to the fetus, what is the nurse's best action? a. Provide the patient with contact information for an AIDS support group. b. Educate the patient about the risks of HIV disease to the fetus. c. Notify the CDC of the patient's diagnosis. d. Provide written and oral education about the use of ART during pregnancy.

1,d

11. What is a primary reason that the normal immune response fails to contain the HIV infection? a. CD4+ T cells become infected with HIV and are destroyed. b. The virus inactivates B cells, preventing the production of HIV antibodies. c. Natural killer cells are destroyed by the virus before the immune system can be activated. d. Monocytes ingest infected cells, differentiate into macrophages, and shed viruses in body tissues.

11. a. Activated CD4+ T cells are an ideal target for HIV because these cells are attracted to the site of concentrated HIV in the lymph nodes, where they become infected through viral contact with CD4 receptors. CD4+ T cells normally are a major component of the immune system and their infection renders the immune system ineffective against HIV and other agents. The virus does not affect natural killer cells and B lymphocytes are functional early in the disease, as evidenced by positive antibody titers against HIV. Monocytes do ingest infected cells and may become sites of HIV replication and spread the virus to other tissue but this does not make the immune response ineffective

11. In a person having an acute rejection of a transplanted kidney, what would help the nurse understand the course of events (select all that apply)? a. A new transplant should be considered. b. Acute rejection can be treated with OKT3. c. Acute rejection usually leads to chronic rejection. d. Corticosteroids are the most successful drugs used to treat acute rejection. e. Acute rejection is common a er a transplant and can be treated with drug therapy.

11. b, e

12. Which characteristic corresponds with the acute stage of HIV infection? a. Burkitt's lymphoma c. Persistent fevers and night sweats b. Temporary fall of CD4+ T cells d. Pneumocystis jiroveci pneumonia

12. b. The symptoms of acute HIV infection occur 2 to 4 weeks after initial infection and last for 1 to 2 weeks. The CD4+ T-cell counts fall temporarily but quickly return to baseline levels. Burkitt's lymphoma and Pneumocystis jiroveci pneumonia are two of the opportunistic diseases that can occur in acquired immunodeficiency syndrome (AIDS). Persistent fevers and drenching night sweats occur in the symptomatic infection stage.

12. What is the most appropriate nursing intervention to help an HIV-infected patient adhere to a treatment regimen? a. "Set up" a drug pillbox for the patient every week. b. Give the patient a video and a brochure to view and read at home. c. Tell the patient that the side e ects of the drugs are bad but that they go away a er a while. d. Assess the patient's routines and nd adherence cues that t into the patient's life circumstances.

12. d

15. Which characteristics describe Pneumocystis jiroveci infection, an opportunistic disease that can be associated with HIV? a. May cause fungal meningitis b. Diagnosed by lymph node biopsy c. Pneumonia with dry, nonproductive cough d. Viral retinitis, stomatitis, esophagitis, gastritis, or colitis

15. c. Pneumocystis jiroveci infection is characterized by pneumonia with a dry, nonproductive cough. Cryptococcus infection may cause fungal meningitis. Non-Hodgkin's lymphoma is diagnosed by lymph node biopsy. Cytomegalovirus infection is characterized by viral retinitis, stomatitis, esophagitis, gastritis, or colitis.

15. Which type of hypersensitivity reaction occurs with rheumatoid arthritis and acute glomerulonephritis? a. Type I or IgE-mediated hypersensitivity reaction b. Type II or cytotoxic hypersensitivity reaction c. Type III or immune-complex mediated hypersensitivity reaction d. Type IV or delayed hypersensitivity reaction

15. c. With rheumatoid arthritis and acute glomerulonephritis, type III or immune-complex mediated hypersensitivity reaction is seen when the antigens combined with IgG and IgM are too small to be removed by the mononuclear phagocytic system and are deposited in tissue, which activates the complement system.

16. Which opportunistic disease associated with AIDS is characterized by hyperpigmented lesions of skin, lungs, and gastrointestinal (GI) tract? a. Kaposi sarcoma c. Herpes simplex type 1 infection b. Candida albicans d. Varicella-zoster virus infection

16. a. Hyperpigmented lesions of skin, lungs, and GI tract are seen in Kaposi sarcoma. Candida albicans is a common yeast infection of the mouth, esophagus, GI tract, or vagina. Herpes simplex type 1 infection has oral and mucocutaneous vesicular and ulcerative lesions. Varicella- zoster virus infection or shingles is a maculopapular, pruritic rash and is treated with acyclovir.

16. For the patient with allergic rhinitis, which therapy should the nurse expect to be ordered first? a. Corticosteroids c. Antipruritic drugs b. Immunotherapy d. Sympathomimetic/decongestant drugs

16. d. The best drugs for allergic rhinitis are antihistamines. However, of those listed, minor sympathomimetic/ decongestant drugs are used primarily for allergic rhinitis. Nasal corticosteroids may be used for seasonal allergic rhinitis; oral corticosteroids are used briefly if the patient is incapacitated. Immunotherapy is used when the allergen cannot be avoided and after it is found that drug therapy is not effective. Antipruritic drugs are topical and used to relieve itching.

17. A patient was given an IM injection of penicillin in the gluteus maximus and developed dyspnea and weakness within minutes following the injection. Which additional assessment findings indicate that the patient is having an anaphylactoid reaction (select all that apply)? a. Wheezing b. Hypertension c. Rash on arms d. Constricted pupils e. Slowed strong pulse f. Feeling of impending doom

17. a, f. Wheezing and a feeling of impending doom can both occur with anaphylaxis. Other common physiologic systemic anaphylactic responses are hypotension; rapid, weak pulse; dilated pupils; and edema and itching at the injection site. An arm rash would be more likely with a simple allergic reaction.

18. The "rapid" HIV antibody testing is performed on a patient at high risk for HIV infection. What should the nurse explain about this test? a. The test measures the activity of the HIV and reports viral loads as real numbers. b. This test is highly reliable, and in 5 minutes the patient will know if HIV infection is present. c. If the results are positive, another blood test and a return appointment for results will be necessary. d. This test detects drug-resistant viral mutations that are present in viral genes to evaluate resistance to antiretroviral drugs.

18. c. Although the "rapid" test is highly reliable and results are available in about 20 minutes, if results are positive from any testing, blood will be drawn for more specific enzyme immunosorbent assay (EIA) or Western blot testing and another visit will be necessary to obtain the results of the additional testing. CD4+ T-cell counts are not used for screening but rather are used to monitor the progression of HIV infection and new assay tests measure resistance of the virus to antiviral drugs.

19. Treatment with two nucleoside reverse transcriptase inhibitors (NRTIs) and a protease inhibitor (PI) is prescribed for a patient with HIV infection who has a CD4+ T-cell count of <400/μL. The patient asks why so many drugs are necessary for treatment. What should the nurse explain as the primary rationale for combination therapy? a. Cross-resistance between specific antiretroviral drugs is reduced when drugs are given in combination. b. Combinations of antiretroviral drugs decrease the potential for development of antiretroviral-resistant HIV variants. c. Side effects of the drugs are reduced when smaller doses of three different drugs are used rather than large doses of one drug. d. When CD4+ T-cell counts are <500/μL, a combination of drugs that have different actions is more effective in slowing HIV growth.

19. b. The use of potent combination antiretroviral therapy limits the potential for development of resistance to antiretroviral medications, the major factor that limits the ability of antiretroviral drugs to inhibit virus replication and delay disease progression. The drugs selected should be ones with which the patient has not been previously treated and that are not cross-resistant with antiretroviral agents previously used by the patient.

19. Which rationale describes treatment of atopic allergies with immunotherapy? a. It decreases the levels of allergen-specific T helper cells. b. It decreases the level of IgE so that it does not react as readily with an allergen. c. It stimulates increased IgG to bind with allergen-reactive sites, preventing mast cell-bound IgE reactions. d. It gradually increases the amount of allergen in the body until it is no longer recognized as foreign and does not elicit an antibody reaction.

19. c. Allergic individuals have elevated levels of IgE that react with allergens to produce symptoms. Immunotherapy involves injecting allergen extracts that will stimulate increased IgG that combines more readily with allergens without releasing histamine. The goal is to keep the "blocking" level of IgG high.

8. Antiretroviral drugs are used to a. cure acute HIV infection. b. decrease viral RNA levels. c. treat opportunistic diseases. d. decrease pain and symptoms in terminal disease.

8. b,

8. A patient is undergoing plasmapheresis for treatment of systemic lupus erythematosus. The nurse explains that plasmapheresis is used in her treatment to a. remove T lymphocytes in her blood that are producing antinuclear antibodies. b. remove normal particles in her blood that are being damaged by autoantibodies. c. exchange her plasma that contains antinuclear antibodies with a substitute fluid. d. replace viral-damaged cellular components of her blood with replacement whole blood.

8. c,

When a patient does not appear for a routine clinic visit, the nurse calls to ask about the missed visit. The patient says, "I don't need to come any longer. I'm so glad I no longer have HIV." The nurse learns that recent laboratory results indicated an "undetectable" HIV viral load and that the patient stopped his medication several weeks earlier. What is the nurse's best response? a. Inform the patient that he must be seen immediately because the undetectable viral load indicates that his medication stopped working. b. Have the patient reschedule his clinic visit. c. Congratulate the patient on his treatment success. d. Educate the patient about the continued need for his medications and ongoing laboratory monitoring.

2,d

2. Which antibiotic-resistant organisms cannot be killed by normal hand soap? a. Vancomycin-resistant enterococci b. Methicillin-resistant Staphylococcus aureus c. Penicillin-resistant Streptococcus pneumoniae d. β-Lactamase-producing Klebsiella pneumoniae

2. a,

2. What accurately describes passive acquired immunity (select all that apply)? a. Pooled gamma globulin b. Immunization with antigen c. Temporary for several months d. Immediate, lasting several weeks e. Maternal immunoglobulins in neonate f. Boosters may be needed for extended protection

2. a, c, d. Passive acquired immunity is received from the injection of gamma globulin, provides immediate immunity, and may last for several weeks or months. Immunization with an antigen and the need for boosters contribute to active acquired immunity. Maternal immunoglobulins in the neonate provide temporary passive innate immunity.

2. One function of cell-mediated immunity is a. formation of antibodies. b. activation of the complement system. c. surveillance for malignant cell changes. d. opsonization of antigens to allow phagocytosis by neutrophils.

2. c,

8. Which immunoglobulin is responsible for the primary immune response and forms antibodies to ABO blood antigens? a. IgA c. IgG b. IgD d. IgM

8. d. IgM immunoglobulin is predominant in the primary immune response and produces antibodies against ABO blood antigens.

20. What is one of the most significant factors in determining when to start antiretroviral therapy in a patient with HIV infection? a. Whether the patient has high levels of HIV antibodies b. Confirmation that the patient has contracted HIV infection c. The patient's readiness to commit to a complex, lifelong, uncomfortable drug regimen d. Whether the patient has a support system to help manage the costs and side effects of the drugs

20. c. Guidelines for initiating antiretroviral therapy (ART) are being updated continuously because of the development of alternative drugs and problems with long-term side effects and compliance with regimens. In the past, ART was always recommended at the time of HIV infection diagnosis but today new guidelines suggest that treatment can be delayed until higher levels of immunosuppression are observed. Whenever treatment is started, an important consideration is the patient's readiness to initiate ART because adherence to drug regimens is a critical component of the therapy.

20. Which description about a nurse who develops a contact dermatitis from wearing latex gloves is accurate? a. This demonstrates a type I allergic reaction to natural latex proteins. b. Use powder-free latex gloves to prevent the development of symptoms. c. Use an oil-based hand cream when wearing gloves to prevent latex allergy. d. This demonstrates a type IV allergic reaction to chemicals used in the manufacture of latex gloves.

20. d. This describes a type IV allergic contact dermatitis that is caused by chemicals used in the manufacturing process of latex gloves. A type I allergic reaction that is a response to the natural rubber latex proteins occurs within minutes of contact with the proteins and may manifest with reactions ranging from skin redness to full-blown anaphylactic shock. Powder-free gloves will avoid respiratory exposure to latex proteins but nonlatex gloves are more helpful. Avoidance of oil-based hand creams when wearing gloves can also help to prevent latex allergic reactions.

21. After teaching a patient with HIV infection about using antiretroviral drugs, the nurse recognizes that further teaching is needed when the patient says a. "I should never skip doses of my medication, even if I develop side effects." b. "If my viral load becomes undetectable, I will no longer be able to transmit HIV to others." c. "I should not use any over-the-counter drugs without checking with my health care provider." d. "If I develop a constant headache that is not relieved with aspirin or acetaminophen, I should report it within 24 hours."

21. b. An undetectable viral load in the blood does not mean that the virus is gone; it is still present in lymph nodes and other organs. Transmission is still possible and use of protective measures must be continued.

21. A 28-year-old male Gulf War veteran tells the nurse he gets a headache, sore throat, shortness of breath, and nausea when his girlfriend wears perfume and when he was painting her apartment. He is afraid he has cancer. What does the nurse suspect may be the patient's problem? a. He has posttraumatic stress disorder. b. He has multiple chemical sensitivities. c. He needs to wear a mask when he paints. d. He is looking for an excuse to break up with his girlfriend.

21. b. Multiple chemical sensitivities are commonly seen with scented products, paint fumes, petroleum products, smoke, pesticides, plastics, and synthetic products and occur in Gulf War veterans. Symptoms vary but include headache, sore throat, breathing problems, nausea as well as pain, skin rash, gastrointestinal (GI) problems, confusion, difficulty concentrating, memory problems, and mood changes. Diagnosis is made based on the patient's health history, as there is no test to diagnose it.

22. Although the cause of autoimmune disorders is unknown, which factors are believed to be present in most conditions (select all that apply)? a. Younger age d. Initiation of autoreactivity by triggers b. Male gender e. Frequent viruses throughout the lifetime c. Inheritance of susceptibility genes

22. c, d. Autoimmune causative factors are genetic susceptibility and initiation of autoreactivity by a trigger. Females and older patients are more likely to develop autoimmune diseases. A viral infection may be a trigger but medications and hormones may also be triggers.

22. Prophylactic measures that are routinely used as early as possible in HIV infection to prevent opportunistic and debilitating secondary problems include administration of a. isoniazid (INH) to prevent tuberculosis b. trimethoprim/sulfamethoxazole (TMP/SMX) for toxoplasmosis c. vaccines for pneumococcal pneumonia, influenza, and hepatitis A and B d. varicella-zoster immune globulin (VZIG) to prevent chickenpox or shingles

22. c. Pneumococcal pneumonia, influenza, and hepatitis A and B vaccines should be given as early as possible in HIV infection while there is still immunologic function. Isoniazid (INH) is used for 9 to 12 months only if a patient has reactive purified protein derivative (PPD) >5 mm, has had high-risk exposure, or has prior untreated positive PPD. Trimethoprim/sulfamethoxazole (TMP/ SMX) is initiated when CD4+ T-cell count is <200/ μL or when there is a history of Pneumocystis jiroveci pneumonia (PCP) and varicella-zoster immune globulin (VZIG) is indicated only after significant exposure to chickenpox or shingles in patients with no history of disease or a negative varicella-zoster virus (VZV) antibody test. Prophylaxis for other opportunistic diseases is noted in eTable 15-2.

23. A patient identified as HIV antibody-positive 1 year ago manifests acute HIV infection but does not want to start antiretroviral therapy at this time. What is an appropriate nursing intervention for the patient at this stage of illness? a. Assist with end-of-life issues b. Provide care during acute exacerbations c. Provide physical care for chronic diseases d. Teach the patient about immune enhancement

23. d. After a patient has positive HIV antibody testing and is in acute disease, the overriding goal is to keep the viral load as low as possible and to maintain a functioning immune system. The nurse should provide teaching regarding ways to enhance immune function to prevent the onset of opportunistic diseases in addition to teaching about the spectrum of the infection, options for care, signs and symptoms to watch for, ways to prevent HIV spread, and ways to adhere to treatment regimens.

24. Before the patient receives a kidney transplant, a crossmatch test is ordered. What does a positive crossmatch indicate? a. Matches tissue types for a successful transplantation b. Determines paternity and predicts risk for certain diseases c. Establishes racial background and predicts risk for certain diseases d. Cytotoxic antibodies to the donor contraindicate transplanting this donor's organ

24. d. A crossmatch mixes recipient serum with donor lymphocytes. A positive crossmatch shows that the recipient has cytotoxic antibodies to the donor and this organ cannot be transplanted without hyperacute rejection occurring. A negative crossmatch indicates that it is safe to do the transplant.

25. A patient with advanced AIDS has a nursing diagnosis of impaired memory related to neurologic changes. In planning care for the patient, what should the nurse set as the highest priority? a. Maintain a safe patient environment b. Provide a quiet, nonstressful environment to avoid overstimulation c. Use memory cues such as calendars and clocks to promote orientation d. Provide written instructions of directions to promote understanding and orientation

25. a. All of the nursing interventions are appropriate for a patient with impaired memory but the priority is the safety of the patient when cognitive and behavioral problems impair the ability to maintain a safe environment.

25. What is the most common cause of secondary immunodeficiency disorders? a. Chronic stress c. Drug-induced immunosuppression b. T-cell deficiency from HIV d. Common variable hypogammaglobulinemia

25. c. Drug-induced immunosuppression with antineoplastic agents and corticosteroids is the most common cause of secondary immunodeficiency. Chronic stress and AIDS, not HIV, may cause secondary immunodeficiency but they are not the most common causes. Primary immunodeficiency is caused by common variable hypogammaglobulinemia.

26. Which characteristics are seen with acute transplant rejection (select all that apply)? a. Treatment is supportive b. Only occurs with transplanted kidneys c. Organ must be removed when it occurs d. The recipient's T cytotoxic lymphocytes attack the foreign organ e. Long-term use of immunosuppressants necessary to combat the rejection f. Usually reversible with additional or increased immunosuppressant therapy

26. d, e, f. Acute transplant rejection occurs when the recipient's T cytotoxic lymphocytes attack the foreign organ. Long-term immunosuppressants help to combat it and it is usually reversible with additional immunosuppression. Treatment of chronic rejection is supportive and irreversible with infiltration of the organ with B and T lymphocytes. Hyperacute rejection occurs when the recipient has antibodies against the donor's human leukocyte antigen (HLA), is most common with kidney transplants, and results in the organ needing to be removed.

27. The patient is experiencing fibrosis and glomerulopathy a year after a kidney transplant. Which type of rejection is occurring? a. Acute c. Delayed b. Chronic d. Hyperacute

27. b. Chronic rejection of a kidney transplant manifests as fibrosis and glomerulopathy (seen with proteinuria, edema, and renal failure), occurs over months or years, and is irreversible. Acute rejection occurs in the first 6 months after transplant. Hyperacute rejection occurs minutes to hours after transplantation and is rare. "Delayed rejection" is not a term used with transplantation.

28. What are the most common immunosuppressive agents used to prevent rejection of transplanted organs? a. Cyclosporine, sirolimus, and muromonab-CD3 b. Prednisone, polyclonal antibodies, and cyclosporine c. Azathioprine, mycophenolate mofetil, and sirolimus d. Tacrolimus, prednisone, and mycophenolate mofetil

28. d. Standard immunotherapy involves the use of three different immunosuppressants that act in different ways:a calcineurin inhibitor (cyclosporine, tacrolimus), a corticosteroid, and the antimetabolite mycophenolate mofetil. Although cyclosporine is still used, tacrolimus is the most frequently prescribed calcineurin inhibitor. Azathioprine (Imuran) is similar to mycophenolate mofetil and they cannot be taken together.

29. The patient has received a bone marrow transplant. Soon after the transplant there is a rash on the patient's skin. She says her skin is itchy and she has severe abdominal pain. What best summarizes what is happening to the patient and how she will be treated? a. Graft rejection occurring; treat with different immunosuppressive agents b. Dry skin and nausea are side effects of immunosuppresants; decrease the dose c. Transplanted bone marrow is rejecting her tissue; prevent with immunosuppressive agents d. Dry skin from the dry air and nausea from the food in the hospital; treat with humidifier and home food

29. c. Graft-versus-host (GVH) disease is occurring as the graft is rejecting the host tissue, which usually manifests in a pruritic or painful skin rash; in the GI tract with diarrhea, severe abdominal pain, GI bleeding, and malabsorption; or in the liver with mild jaundice and elevated liver enzymes. GVH disease is more effectively prevented with immunosuppressive agents than treated.

The nurse advises HIV-positive patients about blood draws for CD4 count. What is the correct information to give them about when and how this laboratory blood work should be done? a. At the same laboratory at approximately the same time of day whenever possible b. After a 10-hour fast c. Approximately 1 hour after taking antiretroviral medications d. At any laboratory at any time of day

3,a

3. The reason newborns are protected for the rst 6 months of life from bacterial infections is because of the maternal transmission of a. IgG. b. IgA. c. IgM. d. IgE.

3. a

3. Transmission of HIV from an infected individual to another most commonly occurs as a result of a. unprotected anal or vaginal sexual intercourse. b. low levels of virus in the blood and high levels of CD4+ T cells. c. transmission from mother to infant during labor and delivery and breastfeeding. d. sharing of drug-using equipment, including needles, syringes, pipes, and straws.

3. a,

3. How does an antigen stimulate an immune response? a. It is captured, processed, and presented to a lymphocyte by a macrophage. b. It circulates in the blood, where it comes in contact with circulating lymphocytes. c. It is a foreign protein that has antigenic determinants different from those of the body. d. It combines with larger molecules that are capable of stimulating production of antibodies.

3. a. Both B and T lymphocytes must be sensitized by a processed antigen to activate the immune response. Processing involves the taking up of an antigen by macrophages, expression of the antigen on the macrophage cell membrane, and presentation to the lymphocytes. Antigens do not need to be proteins and a few antigens may combine with larger molecules that are antigenic.

3. What are the recommended measures to prevent the transmission of health care-associated infections (HAIs) (select all that apply)? a. Empty bedpans as soon as possible b. Limit fresh flowers in patient rooms c. Remove urinals from bedside tables d. Use personal protective equipment e. Hand washing or alcohol-based sanitizing f. Have patients wear sandals in the shower

3. d, e. Hand washing and the use of alcohol-based sanitizers and personal protective equipment (e.g., gloves) will prevent health care-associated infections (HAIs). Although the other interventions will not hurt a patient and they are good practice, they will not prevent HAIs.

In collaboration with the patient on ART, the nurse for- mulates a plan of care. Which items are appropriate to include in planning? (Select all that apply.) a. Patient's viral load will become and remain undetectable. b. Patient will not experience secondary infection. c. Patient will promptly report new onset of symptoms and side effects. d. Patient's laboratory blood work will be within normal limits. e. Patient will adhere to medication regimen and/or report difficulties related to adherence.

4,a,b,c,e

5. the nurse is alerted to possible anaphylactic shock immediately a er a patient has received intramuscular penicillin by the development of a. edema and itching at the injection site. b. sneezing and itching of the nose and eyes. c. a wheal-and- are reaction at the injection site. d. chest tightness and production of thick sputum.

5. a

5. Which statements accurately describe HIV infection (select all that apply)? a. Untreated HIV infection has a predictable pattern of progres- sion. b. Late chronic HIV infection is called acquired immunode - ciency syndrome (AIDS). c. Untreated HIV infection can remain in the early chronic stage for a decade or more. d. Untreated HIV infection usually remains in the early chronic stage for 1 year or less. e. Opportunistic diseases occur more o en when the CD4+ T cell count is high and the viral load is low.

5. a, b, c,

5. A 78-year-old patient has developed Haemophilus influenzae. In addition to standard precautions, what should the nurse use to protect herself and other patients when working within 3 feet of the patient? a. Mask c. Gloves b. Gown d. Shoe covers

5. a. A mask will be worn even 3 feet from the patient to avoid droplet transmission. The gown and gloves will be used as with standard precautions, when working closely with the patient and there is a risk of contamination.

5. How does interferon help the body's natural defenses? a. Directly attacks and destroys virus-infected cells b. Augments the immune response by activating phagocytes c. Induces production of antiviral proteins in cells that prevent viral replication d. Is produced by viral infected cells and prevents the transmission of the virus to adjacent cells

5. c. Interferon produces an antiviral effect in cells by reacting with viruses and inducing the formation of an antiviral protein that prevents new viruses from becoming assembled. Most cytokines are immunomodulatory and do not directly affect antigens, and cytokines such as interleukins may stimulate activation of immune cells.

6. A diagnosis of AIDS is made when an HIV-infected patient has a. a CD4+ T cell count below 200/μL. b. a high level of HIV in the blood and saliva. c. lipodystrophy with metabolic abnormalities. d. oral hairy leukoplakia, an infection caused by Epstein-Barr virus.

6. a,

6. An 82-year-old male patient with pneumonia who is in the intensive care unit (ICU) is beginning to have decreased cognitive function. What should the nurse first suspect as a potential cause of this change? a. Fatigue c. ICU psychosis b. Infection d. Medication allergy

6. b. Infection in older adults often has atypical presentations, cognitive and behavioral changes occurring before fever, pain, or altered laboratory values. Fatigue and ICU psychosis (if the patient is in the ICU) could be occurring but these are not as dangerous for the patient as infection can be. Cognitive and behavioral changes are not typical manifestations of medication allergy.

6. What is included in the humoral immune response? a. Surveillance for malignant cell changes b. Production of antigen-specific immunoglobulins c. Direct attack of antigens by activated B lymphocytes d. Releasing cytokines responsible for destruction of antigens

6. b. Production of immunoglobulins (or antibodies) by B lymphocytes differentiated into plasma cells is the essential component in humoral immunity. Tumor surveillance and the production of cytokines are functions of T lymphocytes in cellular immunity and B lymphocytes do not directly attack antigens.

6. the nurse advises a friend who asks him to administer his allergy shots that a. it is illegal for nurses to administer injections outside of a medical setting. b. he is quali ed to do it if the friend has epinephrine in an injectable syringe provided with his extract. c. avoiding the allergens is a more e ective way of controlling allergies, and allergy shots are not usually e ective. d. immunotherapy should only be administered in a setting where emergency equipment and drugs are available.

6. d,

7. The nurse realizes that the patient understands the teaching about decreasing the risk for antibiotic-resistant infection when the patient says which of the following? a. "I know I should take the antibiotic for one day after I feel better." b. "I want an antibiotic ordered for my cold so I can feel better sooner." c. "I always save some pills because I get the illness again after I first feel better." d. "I will follow the directions for taking the antibiotic so I will get over this infection."

7. d. One of the most important factors in the development of antibiotic-resistant strains of organisms has been inappropriate use of antibiotics. Following directions regarding timing and completion of antibiotics will not allow antibiotic-resistant bacteria to develop. Antibiotics are not effective against viruses, which cause colds and flu. Not completing the antibiotic may allow the hardiest bacteria to survive and multiply and the potential development of an antibiotic-resistant infection.

8. In each of the following situations identify which option has the highest risk for human immunodeficiency virus (HIV) transmission? a. Transmission to women OR to men during sexual intercourse b. Hollow-bore needle used for vascular access OR used for IM injection c. First 2 to 6 months of infection OR 1 year after infection d. Perinatal transmission from HIV-infected mothers taking antiretroviral therapy OR HIV-infected mothers using no therapy e. A splash exposure of HIV-infected blood on skin with an open lesion OR a needle-stick exposure to HIV-infected blood

8. a. women; b. vascular access; c. first 2 to 6 months of infection; d. HIV-infected mothers using no therapy; e. needle-stick exposure to HIV-infected blood

9. the most common cause of secondary immunode ciencies is a. drugs. b. stress. c. malnutrition. d. human immunode ciency virus.

9. a,

9. Which immunoglobulins will initially protect a newborn baby of a breastfeeding mother (select all that apply)? a. IgA d. IgG b. IgD e. IgM c. IgE

9. a, d. IgA is passed to the neonate in the colostrum and IgG crosses the placenta for fetal protection.

9. Opportunistic diseases in HIV infection a. are usually benign. b. are generally slow to develop and progress. c. occur in the presence of immunosuppression. d. are curable with appropriate drug interventions.

9. c,

A patient has acquired immunodeficiency syndrome (AIDS) and the viral load is reported as undetectable. What patient teaching should be provided by the nurse related to this laboratory study result? A The patient has the virus present and can transmit the infection to others. B The patient is not able to transmit the virus to others through sexual contact. C The patient will be prescribed lower doses of antiretroviral medications for 2 months. D The syndrome has been cured, and the client will be able to discontinue all meds.

A

A patient has been diagnosed with human immunodeficiency virus (HIV) infection. What rationale for taking more than one antiretroviral medication should the nurse give to the patient to improve compliance? A Viral replication will be inhibited. B They will decrease CD4+ T cell counts. C It will prevent interaction with other drugs. D More than one drug has a better chance of curing HIV.

A

6. Signs and symptoms of heart transplant rejection include (Select all that apply.) a. SOB. b. tolerance of exercise. c. disturbance in mood. d. decreased weight. e. pulmonary crackles, wheezes. f. onset of hypertension. g. sudden onset of edema.

ANS: A, C, E, G Signs and symptoms of heart transplant rejections include intolerance to exercise, increased weight, and onset of hypotension.

1. A patient who is receiving an IV antibiotic develops wheezes and dyspnea. In which order should the nurse implement these prescribed actions? (Put a comma and a space between each answer choice [A, B, C, D, E]). a. Discontinue the antibiotic infusion. b. Give diphenhydramine (Benadryl) IV. c. Inject epinephrine (Adrenalin) IM or IV. d. Prepare an infusion of dopamine (Intropin). e. Start 100% oxygen using a nonrebreather mask.

ANS: A, E, C, B, D The nurse should initially discontinue the antibiotic because it is the likely cause of the allergic reaction. Next, oxygen delivery should be maximized, followed by treatment of bronchoconstriction with epinephrine administered IM or IV. Diphenhydramine will work more slowly than epinephrine, but will help prevent progression of the reaction. Because the patient currently does not have evidence of hypotension, the dopamine infusion can be prepared last.

3. A patient is admitted to the critical care unit with the diagnosis of massive head trauma caused by a motor vehicle accident. The patient is unresponsive and on mechanical ventilation. The electroencephalogram is negative for brain waves. The family has agreed to organ donation. The organ donor would be classified as a a. brain-dead donor. c. living related donor. b. donation after cardiac death. d. living, unrelated donor.

ANS: A A brain-dead donor is a donor declared dead by neurologic criteria for brain death. Donation after cardiac death is when the donor is declared dead by circulatory criteria for death. A living related donor is a family member related by blood who donates a kidney, portion of the liver, pancreas, intestine, or lung to another family member. A living, unrelated donor (directed or nondirected) is a living individual not related to a patient requiring a transplant who donates a kidney, portion of the liver, pancreas, intestine, or lung to another individual. The donor may be anonymous or altruistic.

18. A patient who has received a transplant is being taught about azathioprine. Which statement made by the patient would indicate the teaching was effective? a. "I will notify my health care provider if my gums start to bleed." b. "I will make sure to increase the amount of fiber in my diet." c. "I realize I may have an increase in hair growth." d. "I know the flulike symptoms will go away as I get used to the drug."

ANS: A A common adverse effect is the suppression of other rapidly proliferating cells, resulting in leukopenia, thrombocytopenia, and anemia. The dose of the drug is adjusted to keep the white blood cell (WBC) count between 3000 and 5000 cells/mm3, thus protecting the patient from an increased risk of infection.

17. A patient is admitted to the hospital with acute rejection of a kidney transplant. Which intervention will the nurse prepare for this patient? a. Administration of immunosuppressant medications b. Insertion of an arteriovenous graft for hemodialysis c. Placement of the patient on the transplant waiting list d. A blood draw for human leukocyte antigen (HLA) matching

ANS: A Acute rejection is treated with the administration of additional immunosuppressant drugs such as corticosteroids. Because acute rejection is potentially reversible, there is no indication that the patient will require another transplant or hemodialysis. There is no indication for repeat HLA testing.

12. A patient has had a kidney transplant. The nurse knows that monitoring which of the following would have the highest priority? a. Fluid volume c. Complete blood count b. Electrolytes d. Temperature

ANS: A Adequate hydration is an absolute necessity for continued graft function in the immediate postoperative period. Hypovolemia can lead to compromised blood flow to the kidney, acute tubular necrosis, and possible graft failure. The new kidney will produce large amounts of urine, and replacement fluids, usually maintained at a ratio of 1:1 mL, must be sustained.

1. A patient who has vague symptoms of fatigue, headaches, and a positive test for human immunodeficiency virus (HIV) antibodies using an enzyme immunoassay (EIA) test. What instructions should the nurse give to this patient? a. "The EIA test will need to be repeated to verify the results." b. "A viral culture will be done to determine the progression of the disease." c. "It will probably be 10 or more years before you develop acquired immunodeficiency syndrome (AIDS)." d. "The Western blot test will be done to determine whether acquired immunodeficiency syndrome (AIDS) has developed."

ANS: A After an initial positive EIA test, the EIA is repeated before more specific testing such as the Western blot is done. Viral cultures are not usually part of HIV testing. It is not appropriate for the nurse to predict the time frame for AIDS development. The Western blot tests for HIV antibodies, not for AIDS.

1. The nurse cares for a patient infected with human immunodeficiency virus (HIV) who has just been diagnosed with asymptomatic chronic HIV infection. Which prophylactic measures will the nurse include in the plan of care (select all that apply)? a. Hepatitis B vaccine b. Pneumococcal vaccine c. Influenza virus vaccine d. Trimethoprim-sulfamethoxazole e. Varicella zoster immune globulin

ANS: A, B, C Asymptomatic chronic HIV infection is a stage between acute HIV infection and a diagnosis of symptomatic chronic HIV infection. Although called asymptomatic, symptoms (e.g., fatigue, headache, low-grade fever, night sweats) often occur. Prevention of other infections is an important intervention in patients who are HIV positive, and these vaccines are recommended as soon as the HIV infection is diagnosed. Antibiotics and immune globulin are used to prevent and treat infections that occur later in the course of the disease when the CD4+ counts have dropped or when infection has occurred.

3. The nurse plans a presentation for community members about how to decrease the risk for antibiotic-resistant infections. Which information will the nurse include in the teaching plan (select all that apply)? a. Continue taking antibiotics until all the medication is gone. b. Antibiotics may sometimes be prescribed to prevent infection. c. Unused antibiotics that are more than a year old should be discarded. d. Antibiotics are effective in treating influenza associated with high fevers. e. Hand washing is effective in preventing many viral and bacterial infections.

ANS: A, B, E All prescribed doses of antibiotics should be taken. In some situations, such as before surgery, antibiotics are prescribed to prevent infection. There should not be any leftover antibiotics because all prescribed doses should be taken. However, if there are leftover antibiotics, they should be discarded immediately because the number left will not be enough to treat a future infection. Hand washing is generally considered the single most effective action in decreasing infection transmission. Antibiotics are ineffective in treating viral infections such as influenza.

5. Potential recipients are matched with donors based on (Select all that apply.) a. blood type. b. human leukocytic antigen. c. race. d. socioeconomic status. e. severity of illness. f. location of recipient. g. waiting time on the list.

ANS: A, B, E, F, G Potential recipients are matched with the donor based on blood type, height, weight, human leukocytic antigen, distance from the donor, waiting time on the list, and severity of illness. The national system in place for organ allocation is fair and equitable for those requiring a transplant. The list does not reference race, gender, or socioeconomic status.

1. Which of the following federal laws regulate the medical and surgical therapy involved with organ transplantation? (Select all that apply.) a. Omnibus Budget Reconciliation Act b. Uniform Anatomical Gift Act c. Hospital Conditions of Participation-Organ Donations d. Medical Examiner Laws e. Uniform Determination of Death Act

ANS: A, C The Uniform Anatomical Gift Act, Uniform Determination of Death Act, and Medical Examiner Laws Restricting Ability of Medical Examiner or Coroner to deny organ donation are all state laws.

1. The nurse provides discharge instructions to a patient who has an immune deficiency involving the T lymphocytes. Which screening should the nurse include in the teaching plan for this patient? a. Screening for allergies b. Screening for malignancy c. Antibody deficiency screening d. Screening for autoimmune disorders

ANS: B Cell-mediated immunity is responsible for the recognition and destruction of cancer cells. Allergic reactions, autoimmune disorders, and antibody deficiencies are mediated primarily by B lymphocytes and humoral immunity.

12. A patient who uses injectable illegal drugs asks the nurse about preventing acquired immunodeficiency syndrome (AIDS). Which response by the nurse is best? a. "Avoid sexual intercourse when using injectable drugs." b. "It is important to participate in a needle-exchange program." c. "You should ask those who share equipment to be tested for HIV." d. "I recommend cleaning drug injection equipment before each use."

ANS: B Participation in needle-exchange programs has been shown to decrease and control the rate of HIV infection. Cleaning drug equipment before use also reduces risk, but it might not be consistently practiced. HIV antibodies do not appear for several weeks to months after exposure, so testing drug users would not be very effective in reducing risk for HIV exposure. It is difficult to make appropriate decisions about sexual activity when under the influence of drugs.

9. The nurse teaches a patient diagnosed with systemic lupus erythematosus (SLE) about plasmapheresis. What instructions about plasmapheresis should the nurse include in the teaching plan? a. Plasmapheresis will eliminate eosinophils and basophils from blood. b. Plasmapheresis will remove antibody-antigen complexes from circulation. c. Plasmapheresis will prevent foreign antibodies from damaging various body tissues. d. Plasmapheresis will decrease the damage to organs caused by attacking T lymphocytes.

ANS: B Plasmapheresis is used in SLE to remove antibodies, antibody-antigen complexes, and complement from blood. T lymphocytes, foreign antibodies, eosinophils, and basophils do not directly contribute to the tissue damage in SLE.

19. Eight years after seroconversion, a human immunodeficiency virus (HIV)-infected patient has a CD4+ cell count of 800/μL and an undetectable viral load. What is the priority nursing intervention at this time? a. Teach about the effects of antiretroviral agents. b. Encourage adequate nutrition, exercise, and sleep. c. Discuss likelihood of increased opportunistic infections. d. Monitor for symptoms of acquired immunodeficiency syndrome (AIDS).

ANS: B + The CD4 level for this patient is in the normal range, indicating that the patient is the stage of asymptomatic chronic infection, when the body is able to produce enough CD4+ cells to maintain a normal CD4+ count. AIDS and increased incidence of opportunistic infections typically develop when the CD4+ count is much lower than normal. Although the initiation of ART is highly individual, it would not be likely that a patient with a normal CD4+ level would receive ART.

6. Which teaching should the nurse provide about intradermal skin testing to a patient with possible allergies? a. "Do not eat anything for about 6 hours before the testing." b. "Take an oral antihistamine about an hour before the testing." c. "Plan to wait in the clinic for 20 to 30 minutes after the testing." d. "Reaction to the testing will take about 48 to 72 hours to occur."

ANS: C Allergic reactions usually occur within minutes after injection of an allergen, and the patient will be monitored for at least 20 minutes for anaphylactic reactions after the testing. Medications that might modify the response, such as antihistamines, should be avoided before allergy testing. There is no reason to be NPO for skin testing. Results with intradermal testing occur within minutes.

23. The health care provider asks the nurse whether a patient's angioedema has responded to prescribed therapies. Which assessment should the nurse perform? a. Ask the patient about any clear nasal discharge. b. Obtain the patient's blood pressure and heart rate. c. Check for swelling of the patient's lips and tongue. d. Assess the patient's extremities for wheal and flare lesions.

ANS: C Angioedema is characterized by swelling of the eyelids, lips, and tongue. Wheal and flare lesions, clear nasal drainage, and hypotension and tachycardia are characteristic of other allergic reactions.

21. Immediately after the nurse administers an intracutaneous injection of an allergen on the forearm, a patient complains of itching at the site and of weakness and dizziness. What action should the nurse take first? a. Remind the patient to remain calm. b. Administer subcutaneous epinephrine. c. Apply a tourniquet above the injection site. d. Rub a local antiinflammatory cream on the site.

ANS: C Application of a tourniquet will decrease systemic circulation of the allergen and should be the first reaction. A local antiinflammatory cream may be applied to the site of a cutaneous test if the itching persists. Epinephrine will be needed if the allergic reaction progresses to anaphylaxis. The nurse should assist the patient to remain calm, but this is not an adequate initial nursing action.

7. The nurse, who is reviewing a clinic patient's medical record, notes that the patient missed the previous appointment for weekly immunotherapy. Which action by the nurse is most appropriate? a. Schedule an additional dose that week. b. Administer the usual dosage of the allergen. c. Consult with the health care provider about giving a lower allergen dose. d. Re-evaluate the patient's sensitivity to the allergen with a repeat skin test.

ANS: C Because there is an increased risk for adverse reactions after a patient misses a scheduled dose of allergen, the nurse should check with the health care provider before administration of the injection. A skin test is used to identify the allergen and would not be used at this time. An additional dose for the week may increase the risk for a reaction.

23. A patient is admitted to the critical care unit with the diagnosis of massive head trauma caused by an MVA. The patient is unresponsive and on mechanical ventilation. EEG is negative for brain waves. The family has agreed to organ donation. Which of the following organs would not be considered for transplant? a. Heart c. Liver b. Lung d. Kidney

ANS: C Brain death is an absolute contraindication for a liver transplantation.

22. The most common viral infection in transplant recipients is a. Aspergillus. c. cytomegalovirus (CMV). b. Candida. d. Pneumocystis jiroveci.

ANS: C CMV is the most common viral infection in transplant recipients. CMV may occur within the first few months after transplantation and may occur later on. Universal prophylaxis and pre-emptive therapy are the main measures taken to prevent occurrence. CMV infection occurs in 25% to 50% of heart transplant recipients but is highest in lung transplant recipients, with a reported incidence of 54% to 92% in those not taking prophylactic antiviral medications. Candida and Aspergillus species infections remain the most common fungal infections in transplant patients. Antimicrobial prophylaxis for Pneumocystis jiroveci with sulfamethoxazole and trimethoprim is used in most centers for successful prevention of this bacterium.

2. A new mother expresses concern about her baby developing allergies and asks what the health care provider meant by "passive immunity." Which example should the nurse use to explain this type of immunity? a. Early immunization b. Bone marrow donation c. Breastfeeding her infant d. Exposure to communicable diseases

ANS: C Colostrum provides passive immunity through antibodies from the mother. These antibodies protect the infant for a few months. However, memory cells are not retained, so the protection is not permanent. Active immunity is acquired by being immunized with vaccinations or having an infection. It requires that the infant has an immune response after exposure to an antigen. Cell-mediated immunity is acquired through T lymphocytes and is a form of active immunity.

2. A patient who has a positive test for human immunodeficiency virus (HIV) antibodies is admitted to the hospital with Pneumocystis jiroveci pneumonia (PCP) and a CD4+ T-cell count of less than 200 cells/μL. Based on diagnostic criteria established by the Centers for Disease Control and Prevention (CDC), which statement by the nurse is correct? a. "The patient meets the criteria for a diagnosis of an acute HIV infection." b. "The patient will be diagnosed with asymptomatic chronic HIV infection." c. "The patient has developed acquired immunodeficiency syndrome (AIDS)." d. "The patient will develop symptomatic chronic HIV infection in less than a year."

ANS: C Development of PCP meets the diagnostic criterion for AIDS. The other responses indicate earlier stages of HIV infection than is indicated by the PCP infection.

7. A young adult female patient who is human immunodeficiency virus (HIV)-positive has a new prescription for efavirenz (Sustiva). Which information is most important to include in the medication teaching plan? a. Driving is allowed when starting this medication. b. Report any bizarre dreams to the health care provider. c. Continue to use contraception while on this medication. d. Take this medication in the morning on an empty stomach.

ANS: C Efavirenz can cause fetal anomalies and should not be used in patients who may be pregnant. The drug should not be used during pregnancy because large doses could cause fetal anomalies. Once-a-day doses should be taken at bedtime (at least initially) to help patients cope with the side effects that include dizziness and confusion. Patients should be cautioned about driving when starting this drug. Patients should be informed that many people who use the drug have reported vivid and sometimes bizarre dreams.

21. A patient is admitted to the critical care unit with the diagnosis of massive head trauma caused by an MVA. The patient is unresponsive and on mechanical ventilation. EEG is negative for brain waves. The family has agreed to organ donation. The nurse turns the patient's head quickly from side to side. The eyes do not move. The nurse knows this is the result of a loss in the a. oculovestibular reflex. c. oculocephalic reflex. b. corneal and jaw reflexes. d. pupillary reflexes.

ANS: C Oculocephalic reflex: Ocular movements are lost with brain death. The oculocephalic reflex, also described as doll's eyes, involves fast turning of the head to both sides. In brain death, this should not generate any eye movements. Oculovestibular reflex: Because the oculovestibular reflex is tested using iced water or normal saline, it is sometimes called cold calorics. The head of the bed is elevated 30 degrees, and approximately 50 mL of ice water or normal saline is injected into the ear; no movement of the eye toward the side of the stimulus should be present. It is recommended that the patient be observed for up to 1 minute after each ear irrigation, and 5 minutes should be allowed before testing the opposite ear. Corneal and jaw reflexes: Facial sensory and motor responses are elicited by testing for corneal and jaw reflexes. Stroking a cotton-tipped swab gently across the cornea tests the corneal reflexes. Pupillary reflexes: Pupillary signs are evaluated by absence of the light reflex, which is consistent with brain death. Most often the pupils are round, oval, or irregularly shaped, although dilated pupils may remain even after brain death has occurred.

10. The nurse palpates enlarged cervical lymph nodes on a patient diagnosed with acute human immunodeficiency virus (HIV) infection. Which action would be most appropriate for the nurse to take? a. Instruct the patient to apply ice to the neck. b. Advise the patient that this is probably the flu. c. Explain to the patient that this is an expected finding. d. Request that an antibiotic be prescribed for the patient.

ANS: C Persistent generalized lymphadenopathy is common in the early stages of HIV infection. No antibiotic is needed because the enlarged nodes are probably not caused by bacteria. Applying ice to the neck may provide comfort, but the initial action is to reassure the patient this is an expected finding. Lymphadenopathy is common with acute HIV infection and is therefore not likely the flu.

24. A nurse has obtained donor tissue typing information about a patient who is waiting for a kidney transplant. Which results should be reported to the transplant surgeon? a. Patient is Rh positive and donor is Rh negative b. Six antigen matches are present in HLA typing c. Results of patient-donor cross matching are positive d. Panel of reactive antibodies (PRA) percentage is low

ANS: C Positive crossmatching is an absolute contraindication to kidney transplantation, since a hyperacute rejection will occur after the transplant. The other information indicates that the tissue match between the patient and potential donor is acceptable.

23. The nurse designs a program to decrease the incidence of human immunodeficiency virus (HIV) infection in the adolescent and young adult populations. Which information should the nurse assign as the highest priority? a. Methods to prevent perinatal HIV transmission b. Ways to sterilize needles used by injectable drug users c. Prevention of HIV transmission between sexual partners d. Means to prevent transmission through blood transfusions

ANS: C Sexual transmission is the most common way that HIV is transmitted. The nurse should also provide teaching about perinatal transmission, needle sterilization, and blood transfusion, but the rate of HIV infection associated with these situations is lower.

8. While obtaining a health history from a patient, the nurse learns that the patient has a history of allergic rhinitis and multiple food allergies. Which action by the nurse is most appropriate? a. Encourage the patient to carry an epinephrine kit in case a type IV allergic reaction to latex develops. b. Advise the patient to use oil-based hand creams to decrease contact with natural proteins in latex gloves. c. Document the patient's allergy history and be alert for any clinical manifestations of a type I latex allergy. d. Recommend that the patient use vinyl gloves instead of latex gloves in preventing blood-borne pathogen contact.

ANS: C The patient's allergy history and occupation indicate a risk of developing a latex allergy. The nurse should be prepared to manage any symptoms that may occur. Epinephrine is not an appropriate treatment for contact dermatitis that is caused by a type IV allergic reaction to latex. Oil-based creams will increase the exposure to latex from latex gloves. Vinyl gloves are appropriate to use when exposure to body fluids is unlikely.

18. The charge nurse is assigning rooms for new admissions. Which patient would be the most appropriate roommate for a patient who has acute rejection of an organ transplant? a. A patient who has viral pneumonia b. A patient with second-degree burns c. A patient who is recovering from an anaphylactic reaction to a bee sting d. A patient with graft-versus-host disease after a recent bone marrow transplant

ANS: C Treatment for a patient with acute rejection includes administration of additional immunosuppressants, and the patient should not be exposed to increased risk for infection as would occur from patients with viral pneumonia, graft-versus-host disease, and burns. There is no increased exposure to infection from a patient who had an anaphylactic reaction.

25. A patient who is receiving immunotherapy has just received an allergen injection. Which assessment finding is most important to communicate to the health care provider? a. The patient's IgG level is increased. b. The injection site is red and swollen. c. The patient's allergy symptoms have not improved. d. There is a 2-cm wheal at the site of the allergen injection.

ANS: D A local reaction larger than quarter size may indicate that a decrease in the allergen dose is needed. An increase in IgG indicates that the therapy is effective. Redness and swelling at the site are not unusual. Because immunotherapy usually takes 1 to 2 years to achieve an effect, an improvement in the patient's symptoms is not expected after a few months.

18. The nurse cares for a patient who is human immunodeficiency virus (HIV) positive and taking antiretroviral therapy (ART). Which information is most important for the nurse to address when planning care? a. The patient's blood glucose level is 142 mg/dL. b. The patient complains of feeling "constantly tired." c. The patient is unable to state the side effects of the medications. d. The patient states, "Sometimes I miss a dose of zidovudine (AZT)."

ANS: D Because missing doses of ART can lead to drug resistance, this patient statement indicates the need for interventions such as teaching or changes in the drug scheduling. Elevated blood glucose and fatigue are common side effects of ART. The nurse should discuss medication side effects with the patient, but this is not as important as addressing the skipped doses of AZT.

9. The nurse will most likely prepare a medication teaching plan about antiretroviral therapy (ART) for which patient? a. Patient who is currently HIV negative but has unprotected sex with multiple partners + b. Patient who was infected with HIV 15 years ago and now has a CD4 840/μL count of c. HIV-positive patient with a CD4+ count of 160/μL who drinks a fifth of whiskey daily d. Patient who tested positive for HIV 2 years ago and now has cytomegalovirus (CMV) retinitis

ANS: D CMV retinitis is an acquired immunodeficiency syndrome (AIDS)-defining illness and indicates that the patient is appropriate for ART even though the HIV infection period is + relatively short. An HIV-negative patient would not be offered ART. A patient with a CD4 count in the normal range would not typically be started on ART. A patient who drinks alcohol heavily would be unlikely to be able to manage the complex drug regimen and would not be appropriate for ART despite the low CD4+ count.

8. A patient who is human immunodeficiency virus (HIV)-infected has a CD4+ cell count of 400/μL. Which factor is most important for the nurse to determine before the initiation of antiretroviral therapy (ART) for this patient? a. HIV genotype and phenotype b. Patient's social support system c. Potential medication side effects d. Patient's ability to comply with ART schedule

ANS: D Drug resistance develops quickly unless the patient takes ART medications on a strict, regular schedule. In addition, drug resistance endangers both the patient and the community. The other information is also important to consider, but patients who are unable to manage and follow a complex drug treatment regimen should not be considered for ART.

16. An older adult patient has a prescription for cyclosporine following a kidney transplant. Which information in the patient's health history has the most implications for planning patient teaching about the medication at this time? a. The patient restricts salt to treat prehypertension. b. The patient drinks 3 to 4 quarts of fluids every day. c. The patient has many concerns about the effects of cyclosporine. d. The patient has a glass of grapefruit juice every day for breakfast.

ANS: D Grapefruit juice can increase the toxicity of cyclosporine. The patient should be taught to avoid grapefruit juice. High fluid intake will not affect cyclosporine levels or renal function. Cyclosporine may cause hypertension, and the patient's many concerns should be addressed, but these are not potentially life-threatening problems.

7. Rejection that occurs within hours after the transplantation and results in immediate graft failure is known as a. acute rejection. c. chronic rejection. b. intermediate rejection. d. hyperacute rejection.

ANS: D Hyperacute rejection is a humoral-mediated response, which occurs within hours after transplantation and results in immediate graft failure. Acute rejection tends to occur weeks to months after transplantation but can occur at any time. Chronic rejection occurs at varying times after transplantation and progresses for years until it leads to ultimate failure of the transplanted organ.

4. A patient is admitted to the critical care unit with the diagnosis of massive head trauma caused by a motor vehicle accident. The patient is unresponsive and on mechanical ventilation. The electroencephalogram is negative for brain waves. The family has agreed to organ donation. The nurse will check for the absence of cerebral motor reflexes by a. performing a sternal rub. b. needle pricks to fingers and toes. c. clapping hands near the ears. d. applying pressure to the nail beds or supraorbital ridge.

ANS: D These motor responses can be stimulated by the application of pressure to the nail beds or supraorbital ridge.

17. A patient who has received a transplant is being taught about cyclosporine. Which statement made by the patient would indicate the teaching was effective? a. "I know this drug prevents my immune system from working." b. "If I find the capsules are hard to swallow, I'll take the liquid." c. "I will need to watch for bruising." d. "I will need to monitor my blood pressure."

ANS: D This drug can cause hypertension. Capsules and liquid form are not interchangeable. The immune system still has some ability to work. Cyclosporine does not affect the bone marrow.


Ensembles d'études connexes

AP Statistics Probability Formulas

View Set

Pharm Chapter 10 - Antiviral Agents

View Set

NUAS240T - Chapter 19 - Nursing Management of Pregnancy at Risk: Pregnancy-Related Complications

View Set